What is Mistake: Definition and 295 Discussions

In contract law, a mistake is an erroneous belief, at contracting, that certain facts are true. It can be argued as a defense, and if raised successfully can lead to the agreement in question being found void ab initio or voidable, or alternatively an equitable remedy may be provided by the courts. Common law has identified three different types of mistake in contract: the 'unilateral mistake', the 'mutual mistake' and the 'common mistake'. The distinction between the 'common mistake' and the 'mutual mistake' is important.
Another breakdown in contract law divides mistakes into four traditional categories: unilateral mistake, mutual mistake, mistranscription, and misunderstanding.The law of mistake in any given contract is governed by the law governing the contract. The law from country to country can differ significantly. For instance, contracts entered into under a relevant mistake have not been voidable in English law since Great Peace Shipping v Tsavliris (International) Ltd (2002).

View More On Wikipedia.org
  1. S

    Intensity of central maximum when width of single slit is doubled

    My answer is (D) but the correct answer is (B). I thought the intensity will be 4 times since the slit width is doubled and amplitude becomes twice. What is my mistake? Thanks
  2. murshid_islam

    I What's my mistake in this integration problem?

    Here's the problem: ##\int_0^{2\pi} \cos^{-1}(\sin(x)) \mathrm{d}x## If I do the substitution ##u = \sin(x)##, both the limits of integration become 0 and the integral would result in 0. But the graph of the function tells me the area isn't 0. Where am I going wrong?
  3. MatinSAR

    Prove that K is a tensor using quotient theorem

    $$K'_{ij}A'_{jk}=B'_{ik}=a_{ip}a_{kq}B_{pq}=a_{ip}a_{kq}K_{pr}A_{rq}=a_{ip}a_{kq}K_{pr}a_{jr}a_{kq}A'_{jk}$$$$K'_{ij}=a_{ip}a_{kq}a_{kq}a_{jr}K_{pr}$$ Can someone point out my mistake? What I've found shows that K is not a tensor. It is different from my book and I cannot find my mistake...
  4. S

    Question about permutation formula -- How to use the formula when r = 0 ?

    ##P^5_0=\frac{5!}{(5-0)!}=1## But when I use ##P^n_r=n(n-1)...(n-r+1)##, I get ##P^5_0=5(4)...(6)## Where is the mistake? Thanks
  5. S

    My Mistake? Understanding Friction Force & Work Done on Snowy & Icy Surfaces

    The answer is (D), but I don't understand why. Option (A) is wrong because the work done = 0. Then, I divide the motion into 3 parts: 1) motion on snowy surface Since the sledge is being pulled horizontally (let assume to the right), there will be tension force T to the right and friction...
  6. DioMiner

    Why does this not work? Projectile Motion problem for Dynamics Class

    TL;DR Summary: I want to mainly figure out where in the problem solving I went wrong. I understand the correct answer (since I looked it up), but to me, it does not make any sense. I am honestly stumped at this point. Online solutions say that my equation y = 0.5774x-0.003354x^2 should...
  7. S

    Finding a Mistake: Probability of Y with X and Spins

    I divide the question into three cases: 1) P (Y = 1 and X = 5 or 7) = 1/5 + 1/5 = 2/5 2) P(Y = 2 and 1st spin = even and 2nd spin = 7) = 2/5 x 1/5 = 2/25 3) P(Y = 3 and 1st and 2nd spin = even and 3rd spin = 7) = 2/5 x 2/5 x 1/5 = 2/125 Total probability = 2/5 + 2/25 + 2/125 = 62/125 But...
  8. brotherbobby

    Inequality involving a reciprocal - where's the mistake?

    Problem Statement : Solve the inequality : ##\left( \dfrac{1}{3} \right)^x<9##. Attempts: I copy and paste my attempt below using Autodesk Sketchbook##^{\circledR}##. The two attempts are shown in colours black and blue. Issue : On checking, the first attempt in black turns out to be...
  9. H

    How to never make a mistake with the object of a verb?

    What can I do so as to never make a mistake regarding the object of a verb, that is whether the verb will take an object or not (transitive/intransitive) and if it takes then how many are allowed? Learning all the verbs in each category doesn’t seem to be a very good idea. General knowledge...
  10. D

    Solving a Problem: Have I Made a Mistake or Are the Solutions Wrong?

    Hi everyone To solve the below problem, I assumed the affected area was 2x2 minus the definite integral of the given function between 2 and 4. I then equated the answer for that with the given function to solve for a, b and c. I don't know why the solutions give b as 2ln5. Have I made a...
  11. shivajikobardan

    Comp Sci Mistake while removing implications in first order predicate logic-:

    As you can see I am not getting correct result. What have I messed up? I want to learn it. https://slideplayer.com/slide/4942120/ Here is full slide in case anyone wants to refer to it.
  12. E

    Programs I chose math instead of physics. Was it a mistake?

    Long story short, my career goals are to work as a researcher/research scientist in machine learning or to develop models/algorithms for forecasting such as in financial markets or otherwise. Fundamentally, I find think this is quite similar to physics as a science, but that is for another...
  13. W

    B Symmetry of parity: Mistake in the experiment?

    Recently I saw this YouTube video from Veritassium about CPT -Symmetry: In this video an experiment of Prof. Chien-Shiung Wu is presented, which has proven that parity is not symmetric, by observing the emmition of electrons from Co60 atoms with synchronised spin. After thinking about this...
  14. kyphysics

    If I Made Mistake on Taxes in 2019, Can I Correct It If..

    it was a situation that would have led to me getting some of the money back (i.e., I paid too much that year)? I know if you paid too little, the IRS would want you to correct it obviously, so they can rake in that $. But, what if you made an error and should have paid less that year? Would...
  15. R

    Engineering Thermal expansion coefficient calculation — where is my mistake?

    Hello everyone, Once I got through the VDW state equation I came to the expression of the thermal expansion coefficient. When I place the values I get an illogical answer. Is there a problem with the units? (Please ignore the values) Thanks. This is the unit equation I get to and get stuck:
  16. Leo Liu

    A mistake in the derivation of escape velocity

    In the last step of the derivation of escape velocity, the two sides of the equation seem to have opposite signs. $$-1/2mv_0^2=-mgR_e^2\,\lim_{r\to\infty}(1/r-1/R_e)$$ $$-1/2mv_0^2=mgR_e^2 \frac{1}{R_e}$$ Since the mass and the square of the velocity are positive, the left side of the equation...
  17. Adesh

    What's my mistake in this problem in dynamics involving pulleys?

    Find the acceleration of block A and B, given that the mass of pulleys and strings are negligible.. I could see that the block B has gravity acting on it, therefore the gravitational force on the block ##B## is ##F_B = 5 g## and hence the acceleration is ##g##. From the pulley to which...
  18. S

    Finding a Mistake in My Sketch: Can I Get Emergent Beam Perpendicular to AB?

    This is my sketch: I think the only possible way to get emergent beam perpendicular to AB is when the incoming ray is refracted towards side AC then total internal reflection occurs as shown in my sketch. Angle QAR = 60o and angle ARQ = 90o so angle AQR = 30o It means angle CQP is also 30o...
  19. E

    I can't find my mistake in using the chain rule here

    I literally don't know what's going wrong today, I can't seem to get anything right :oldconfused:. The velocity in S' is easy enough $$v' = \frac{dx'}{dt'} = \frac{\partial f}{\partial t} \frac{\partial t}{\partial t'} + \frac{\partial f}{\partial x}\frac{\partial x}{\partial t}\frac{\partial...
  20. U

    I Mistake in problem statement?

    Is it just me, or is the statement simply untrue? I have created a simple counterexample that is consistent with the hypotheses, but results in a contradiction of the result. It is included in pictorial form as the second attachment. Any insights?
  21. agnimusayoti

    What is my mistake in solving for this limit?

    I tried to substitution n = infinity so I got (infinity)*sin (1/infinity). I thought 1/infinity is approaching zero. So, sin (1/infinity) is same with sin (0). With these idea, my solution is lim n--> infinity n sin(1/n) = 0. But, the answer book say that the answer is 1. I tried another...
  22. E

    Chemistry Identifying the mistake in this reaction scheme to form 2-hydroxyisobutyramide

    The steps are as follows (apologies for my awful formatting, I'm not sure which Chemistry latex package there is!), $$(CH_3)_2CO \xrightarrow{HCN} (CH_3)_2C(OH)CN \xrightarrow{H_2SO_4} (CH_3)_2C(OH)COOH \xrightarrow{PCl_5} (CH_3)_2C(OH)COCl \xrightarrow{NH_3} (CH_3)_2C(OH)CONH_2$$ As far as I...
  23. jim mcnamara

    "Fatal" mistake for flat earther

    "Mad Mike" Hughes wanted to see for himself if the Earth was a sphere or flat. Went up in a rocket, when it failed, he ejected without his parachute and died in the fall. https://www.polygraph.info/a/flat-earth-believer-mad-mike-hughes-dies-in-diy-rocket-crash/30454704.html Just think how...
  24. MathematicalPhysicist

    A A possible mistake in Equations (18.204)-(18.205) in Peskin & Schroeder

    They write the following on page 646: Now, equation (17.17) reads: ##\alpha_s(Q) = \frac{2\pi}{b_0 \log(Q/\Lambda)}##, so if I plug ##\alpha_s(Q^2) = \frac{2\pi}{b_0 \log(Q^2/\Lambda)}## into Eq. (18.204) I get: ##\frac{a^f_n}{4b_0}\frac{1}{\log(Q^2/\Lambda)}M^-_{fn}##. Perhaps the...
  25. J

    I Is There a Mistake in This CLQG Thesis?

    I've been reading this interesting thesis recently posted. But I think I spotted a mistake. On page 52 of "Semi-Classical Holomorphic Transition Amplitudes in Covariant Loop Quantum Gravity": https://arxiv.org/pdf/2001.04651.pdf It says "In general, removing ##k## vertices from ##[v_0, v_1, ...
  26. S

    MHB Possible mistake in question? (system of linear inequalities)

    Hello, so there's a graph provided in the task which I'm trying to solve for a quite a while and I am really confused where the 5 to 8 line came from, because (5x+8y<5) doesn't create that sort of line. Is it possible that there's a misstake done by my teacher or am I understanding something wrong?
  27. I

    Simple Finite Square Well Problem help *Ignore, made stupid mistake*

    I've tried to carry out the solution to this as a normal 2nd order Differential Equation ##\psi ##'' - ##-k^2 \psi ## = 0 Assume solution has form ##e^{\gamma x}## sub this in form ##\psi## and get ##\gamma ^2## ##e^{\gamma x} ## + ##k^2 e^{\gamma x}## = 0 Solution is ##\gamma## = 0 or ##k^2##...
  28. Efeguleroglu

    Where's my mistake? (Pendulum with gravity)

    I have a pendulum and an object with radius "R" and mass "m". There are forces: constant gravitational acceleration and tension on the rope. I can write: $$x=R sin(\theta) \ \ y=R cos(\theta)$$ $$\dot{x}=R\dot{\theta}cos(\theta) \ \ \dot{y}=-R\dot{\theta}sin(\theta)$$...
  29. Efeguleroglu

    Where's my mistake? (Lorentz Transformation for a moving spaceship)

    That's what I found. But the answer is arctan(sinθ*sqrt(1-v^2/c^2)/(cosθ+v/c))
  30. SamRoss

    I Missing exponent in "Theoretical Minimum"?

    In "The Theoretical Minimum" (the one on classical mechanics), on page 218, the authors write a Lagrangian $$L=\frac m 2 (\dot r^2 +r^2\dot \theta ^2)+\frac {GMm} r$$ They then apply the Euler-Lagrange equation ##\frac d {dt}\frac {dL} {d\dot r}=\frac {dL} {dr}## (I know there should be...
  31. J

    I Mistake in Schaum's Group Theory?

    Schaum's Outline of Group Theory, Section 3.6e defines {{\rm{L}}_n}\left( {V,F} \right) as the set of all one to one linear transformations of V, the vector space of dimension n over field F. It then says "{{\rm{L}}_n}\left( {V,F} \right) \subseteq {S_V}, clearly". ({S_V} here means the set...
  32. EEristavi

    Linear Momentum - Bullet hitting pendulum

    Solving using Linear Momentum: M vb2/2 = M g 2L vb = 2√(g L) m v = m v/2 + M (2√(g L) ) v = 4 M √(g L) / m Note: I see from the answers - that this is correct. -------------- Next, I tried to solve it via Energy conservation point of view. M vb2/2 = M g 2L vb = 2√(g L) m v2/2 = m v2/8 + k...
  33. EEristavi

    Object causing another object to move with zero friction

    Homework Statement This is the problem from the book "physics for scientists and engineers..." by Serway, Jewett / Chapter 5, problem 98 Initially, the system of objects shown in Figure P5.93 - is held motionless. The pulley and all surfaces and wheels are frictionless. Let the force F be...
  34. tomdodd4598

    I Frauchiger & Renner: There's a Mistake, Right?

    Hey there! I was recently pointed to this thought experiment, claiming an apparent 'contradiction' involving the various predictions of the observers. Now, this has been discussed on PF quite recently, but I found the discussion rather hard to follow. I've read the paper, the PF discussion...
  35. E

    Where is my mistake? (Pure rolling and center of mass)

    Homework Statement A cylinder with a mass of M and radius R is attached to a spring with a constant of k. The attachment always stays the same as described in the picture. The cylinder is moved to the right and the released, and it rolls without slipping (pure rolling). What will be the...
  36. W

    Relations on Sets: Need help understanding a mistake

    Homework Statement Suppose ##R## and ##S## are relations on a set ##A##. If ##R## and ##S## are transitive, is ##R \cup S## transitive? Why? Homework EquationsThe Attempt at a Solution Suppose that ##a## is an arbitrarily but particularly picked element of ##R \cup S##, then $$a \in R \...
  37. beefbrisket

    I Sign mistake when computing integral with differential forms

    The question provides the vector field (xy, 2yz, 3zx) and asks me to confirm Stokes' theorem (the vector calc version) but I am trying to use the generalized differential forms version. So, I am trying to integrate \omega = xy\,dx + 2yz\,dy + 3zx\,dz along the following triangular boundary...
  38. E

    Was there a mistake in our final exam question about resistance?

    Homework Statement This graph was given, which describes the current through a resistor, as a function of the voltage. Then, we were supposed to find the resistance of the resistor when 3V<V<5V. But it changes throughout this range, doesn't it? Because in the answers (Mind you, this is the...
  39. nomadreid

    B Mistake in my complex exponentiation: where?

    I am sure I am overlooking something elementary, but playing around with exponentiation (this is not an assignment), I seem to be making a mistake somewhere. Please don't send me a link for a more compact way of getting the correct result; I wish to know what my particular mistake is. Suppose...
  40. BillTre

    Big Mistake in Operating Room, Patient Dead

    The patent in Russia being operated on for ovarian cysts had a formalin (37% formaldehyde solution) mistakenly introduced into her abdomen. It was washed out when the mistake was recognized, but ultimately, too late. Formaldehyde is used for many things, in biology it is often used to fix...
  41. J

    Deriving gravitational potential energy -- mistake

    Homework Statement Hi I'm attempting to derive the gravitational potential energy of a point mass (##m##) that's moving from infinity to a point r' inside a gravitational field produced by a another mass ##M##. For simplicity I treated it as a one dimensional case. The problem I get is that the...
  42. K

    I Is this a mistake in Weinberg's book?

    The image is below. My question is, why he considers ##p' = p## on the RHS of ##(2.5.14)## but not on the LHS, i.e., the sub index on the left ##\Psi## remains ##p'##. Is it a mistake? And he continues on the derivation (not shown here) even considering ##|N(p)|^2## instead of ##N(p')^*N(p)## so...
  43. mishima

    B Absolute Value Inequality, |x|>|x-1|....where's my mistake?

    Rule: Suppose a>0, then |x|>a if and only if x>a OR x<-a So |x|>|x-1| becomes: x>x-1 which is false (edit: or more accurately doesn't give the whole picture, it implies true for all x) OR x<-x+1 2x<1 x<1/2 which is false
  44. MichPod

    B Where is the mistake -- violation of Heisenberg uncertainty

    The following is kind of a semiclassical reasoning which goes along the style of the discussions between Bohr and Einstein in Solvay Conference. Suppose we have a single slit from which a particle may be emitted and, on a very large distance from it, a thin cloud chamber CC, which we use as a...
  45. Hawksteinman

    I Differentiation of sin function where's my mistake?

    I was thinking and came up with this. I know it's wrong but can't find the mistake :( dy/dx sin(x) = cos(x) dy/dx sin(kx) = kcos(kx) So dy/dx sin(3x) = 3cos(3x) Now let Y = 3x dy/dx sin(Y) = cos(Y) = cos(3x) 3cos(3x) = cos(3x) 3 = 1 Where is the mistake?
  46. D

    Calculation of Speed for Message Packet on Small Planet

    Homework Statement The residents of a small planet have bored a hole straight through its center as part of a communications system. The hole has been filled with a tube and the air has been pumped out of the tube to virtually eliminate friction. Messages are passed back and forth by dropping...
  47. L

    Is this a mistake on the answer key? (Newton's Laws)

    Homework Statement Homework Equations F = ma [/B]The Attempt at a Solution Why is the angle 60 degrees? Isn't it 30 degrees? [/B]
  48. W

    Thermodynamics: Comment on mistake

    Homework Statement Suppose you have a mole of water at 25##^{\circ}## at atmospheric pressure. You then raise its temperature to 30##^{\circ}##. Determine the change in Gibbs free energy. To compensate for the change, once can raise the pressure on water, how much pressure you need to increase...
  49. A

    Is math homework a mistake for pre-teen and teenage students?

    Most people don't get to cal 1 in high-school by the time they are seniors. People are excepted to get ready for cal 2 by the time they get to college. People don't get to cal 1 because they don't study at their home. However, in my personal opinion, is homework a mistake to give to students...
  50. G

    Engineering DC circuit analysis: conceptual mistake?

    Homework Statement Homework Equations[/B] I have a circuit that consists on 3 components in parallel: a current source Ia, a resistor R and a capacitor C. Ia is directed upwards, while the currents through the capacitor and the resistor are both directed downwards. Select the correct option in...
Back
Top